Добавил:
Upload Опубликованный материал нарушает ваши авторские права? Сообщите нам.
Вуз: Предмет: Файл:
Lektsia_2-3.doc
Скачиваний:
16
Добавлен:
10.12.2018
Размер:
1.39 Mб
Скачать

Вопросы существования пределов.

Определение. Пусть дана числовая последовательность x1,x2,…,xn,… .

Эта последовательность называется фундаментальной, если для любого сколь угодно малого >0 найдется номер N=N(), зависящий от , такой, что начиная с этого номера, т.е.

Фундаментальная последовательность называется так же сходящейся «в себе».

Пример. 1) Покажем, что последовательность - фундаментальна.

Доказательство.

Возьмем N=+1,т.е. N>. Тогда

2) Покажем, что последовательность 0,1,0,1,… не является фундаментальной.

Какой бы номер N мы не взяли, найдутся такие числа n, m=n+1, что , т.е. последовательность не фундаментальна.

Свойства фундаментальной последовательности.

Теорема 1. Фундаментальная последовательность ограничена.

Доказательство. Пусть {xn} – фундаментальная последовательность. Покажем, что

Т.к. {xn} – фундаментальная последовательность, то

>0 N=N():

Возьмем =1, тогда =1

В частности,

Если , то

Положим С=, тогда n=1,2,…, т.е. фундаментальная последовательность ограниченна. ч.т.д.

Теорема 2. (Критерий Коши) (Критерий – необходимое и достаточное условие – сходимости).

Для того, чтобы числовая последовательность имела предел, необходимо и достаточно, чтобы она была фундаментальна.

Доказательство.

1. Необходимость. Дано: {xn}-сходящаяся, xn→a, n→.

Доказать, что {xn}- фундаментальная.

Рассмотрим (1)

Т.к. {xn}-сходящаяся, то возьмем >0, тогда

Если n, mN, то виду (1) имеем ч.т.д.

2. Достаточность. Дано: {xn}- фундаментальная, доказать, что {xn}- сходящаяся.

Т.к. {xn}-фундаментальна, то она ограничена.

Пусть I1 – наименьший отрезок, содержащий все члены последовательности.

I2 – наименьший отрезок, содержащий все члены {xn}, начиная со 2-го: х2, х3,…

И.т.д.

Ik - наименьший отрезок, содержащий все члены {xn}, начиная с k-го xk,xk+1,…

И т.д.

Тогда I1I2I3….

Согласно лемме о вложенных отрезках, найдется точка а, принадлежащая всем этим отрезкам:

Т.к. {xn}- фундаментальная, то возьмем >0, тогда

В частности, если m=N, то .

В частности это означает, что в интервал попадает отрезок IN (т.е. элементы xn ).

Т.о. имеем, что точка а и все члены последовательности для содержатся в интервале длины , т.е. , т.е. xn→a, n→. Ч.т.д.

Примеры (с.71 и с.73)

1) Пользуясь критерием Коши, доказать сходимость последовательности {xn}, где xn=1+++…+.

Доказательство. Возьмем >0 и рассмотрим разность . Имеем

==++…+<++

+…+=+++…+=

=-<

Т.о. получили, что pN <.

Рассмотрим неравенство <n>. Положим N=, тогда n>N будет <, следовательно, n>N и pN <.

Следовательно, данная последовательность сходится. Ч.т.д.

2) Пользуясь критерием Коши, доказать расходимость последовательности {xn}, где xn=1+++…+.

Доказательство. Возьмем любое , удовлетворяющее условию 0<< и рассмотрим разность . Имеем

=++…+.

В правой части р слагаемых, - наименьшее из этих слагаемых. Если каждое слагаемое в правой части заменить на наименьшее, то получим >, откуда при p=n будем иметь >> n. Следовательно, {xn} расходится. Ч.т.д.

Соседние файлы в предмете [НЕСОРТИРОВАННОЕ]